LSAT and Law School Admissions Forum

Get expert LSAT preparation and law school admissions advice from PowerScore Test Preparation.

 Administrator
PowerScore Staff
  • PowerScore Staff
  • Posts: 8917
  • Joined: Feb 02, 2011
|
#25099
Complete Question Explanation
(The complete setup for this game can be found here: lsat/viewtopic.php?t=4751)

The correct answer choice is (E)

The question stem indicates that Template #1 is in effect, because only Template #1 allows T to test G on the second day. In that Template, if T tests G on the second day, then only H remains for Y to test on the second day. Consequently, answer choice (E) is correct.
 anahi78
  • Posts: 16
  • Joined: Nov 09, 2013
|
#12879
In the explanation, I see that template #1 doesn't include H with S. It show S with GJ only. Why? Also for #14, if it must be true, then shouldn't the answer have only one option. In this case, Y2 could have been H or G?

Please clarify. Thanks! Anahi
User avatar
 KelseyWoods
PowerScore Staff
  • PowerScore Staff
  • Posts: 1079
  • Joined: Jun 26, 2013
|
#12882
Hi Anahi!

Template 1 is what happens if F is in Y1 and S2 and H is in T1. Since those are the restrictions used to make that template, that means that H has to be in T1 and then can't be in S2 because F is there.

For question #14, if T tests G on the 2nd day, we have to be in Template #1 because that's the only Template where it's possible for G to be in T2. But if T tests G on the 2nd day, then Y no longer has the option of H or G on the 2nd day. Y would have to test H on the 2nd day because T already took G for the 2nd day.

Hope that helps!

Best,
Kelsey
User avatar
 jojgreen
  • Posts: 4
  • Joined: Apr 09, 2021
|
#86251
Hello,

Do you mind explaining why answer B is incorrect? I was stuck between answer choices B and E, and found that answer choice B could work, but I wasn't sure why E must be true versus B. Thank you!
User avatar
 KelseyWoods
PowerScore Staff
  • PowerScore Staff
  • Posts: 1079
  • Joined: Jun 26, 2013
|
#86301
Hi jojgreen!

If T tests G on day 2, that means that T must test H on day 1 (because she has to test H on one of the days). Since Y's day 1 has to be the same as S's day 2, that means that the only bike left that they can have in common is F (because Y can't test H on day 1, S can't test G on day 2, and Y can't test J). Since Y can't test H, the only bike left for Y on day 2 is H. That leaves J for R on day 2, which means R must test G on day 1 and S must test J on day 1. Therefore, R cannot test J on the first day.

Here's the diagram:

Y:  F   H
T:  H   G
S:  J    F
R:  G   J
     1     2

Hope this helps!

Best,
Kelsey

Get the most out of your LSAT Prep Plus subscription.

Analyze and track your performance with our Testing and Analytics Package.